From f7bd6359bd1524821c40b4d2902eab7e328245f1 Mon Sep 17 00:00:00 2001 From: Josia Pietsch Date: Mon, 23 Oct 2023 11:44:15 +0200 Subject: [PATCH] lecture 02 --- inputs/lecture_02.tex | 128 ++++++++++++++++++++++++++++++++++++++++++ logic2.tex | 3 +- 2 files changed, 130 insertions(+), 1 deletion(-) create mode 100644 inputs/lecture_02.tex diff --git a/inputs/lecture_02.tex b/inputs/lecture_02.tex new file mode 100644 index 0000000..331efb9 --- /dev/null +++ b/inputs/lecture_02.tex @@ -0,0 +1,128 @@ +\lecture{02}{2023-10-19}{Topology on $\R$} +\begin{definition} + A set $O \subseteq \R$ is called \vocab{open} in $\R$ iff it is the union + of a set of open intervals. + + A set $A \subseteq \R$ is called \vocab{closed} in $\R$ + iff it is the complement of an open set. +\end{definition} + +\begin{remark} + \begin{itemize} + \item If $\emptyset \neq O \R$ + then $O \sim \R$. + \item If $O \subseteq \R$ + is open, then $O$ is the union of open intervals + with rational endpoints, since $\Q$ is dense. + \end{itemize} +\end{remark} + +\begin{remark}+ + $\{O \subseteq \R\} \sim 2^{\aleph_0} < \cP(\R)$. +\end{remark} + +\begin{definition} + We call $x \in \R$ + an \vocab{accumulation point} + of $A$ iff for all $a < x < b$ + there is some $y \in A$, $y \in (a,b)$, $y \neq x$. + We write \vocab{$A'$} for the set of all accumulation points of $A$. +\end{definition} +\begin{example} + $\{\frac{1}{n+1} | n \in \N\}' = \{0\}$. +\end{example} + +\begin{lemma} + \label{lem:closedaccumulation} + A set $A \subseteq \R$ is closed iff $A' \subseteq A$. +\end{lemma} +\begin{refproof}{lem:closedaccumulation} + ``$\implies$'' + Let $A$ be closed. Suppose that $x \in A' \setminus A$. + Then there exists $(a,b) \ni x$ + disjoint from $A$. Hence $x \not\in A' \lightning$ + + ``$\impliedby$'' + Suppose $A' \subseteq A$. + \begin{claim} + $A \subseteq \R$ is closed iff all Cauchy sequences + in $A$ converge in $A$. + \end{claim} + \begin{subproof} + Let $A$ be closed and $\langle x_n : n \in \omega \rangle$ + a Cauchy sequence in $A$. + Suppose that $x = \lim_{n \to \infty} x_n \not\in A$. + Then there is $(a,b) \ni x$ disjoint from $A$. + However $x_n \in (a,b)$ for almost all $n \in \omega$ $\lightning$ + + On the other hand let $A$ not be closed. + Then there exists a witness $x \in \R \setminus A$ + such that $A \cap (a,b) \neq \emptyset$ for all $(a,b) \ni x$. + In particular, + we may pick $x_n \in (x - \frac{1}{n+1}, x + \frac{1}{n+1}) \cap A$ + for all $n < \omega$. + \end{subproof} + + Now if $A' \subseteq A$ and $A$ were not closed, + there would be some Cauchy-sequence $(x_n)$ + in $A$ such that $\lim_{n \to \infty} x_n \not\in A$. + But then $x \in A' \subseteq A \lightning$. +\end{refproof} +\begin{definition} + $P \subseteq \R$ is called \vocab{perfect} + iff $P \neq \emptyset$ and $P = P'$. +\end{definition} + +We want to prove two things: +\begin{itemize} + \item If $P$ is perfect, then $P \sim \R$. + \item If $A$ is closed and uncountable then $A$ has a perfect subset. + In particular $A \sim \R$. +\end{itemize} + +\begin{lemma} + Let $P \subseteq \R$ be perfect. + Then $P \sim \R$. +\end{lemma} +\begin{proof} + It suffices to find an injection $f\colon \R \hookrightarrow P$. + We have $\underbrace{\{0,1\}^{\omega}}_{\text{infinite 0-1-sequences}} \sim \R$, + hence it suffices to construct $f\colon \{0,1\}^\omega\hookrightarrow P$. + + In order to do that, we are going to construct some + $g\colon \underbrace{\{0,1\}^{<\omega}}_{\text{finite 0-1-sequences}} \to P$ + with certain properties + by recursion on the length of $s \in \{0,1\}^{<\omega}$. + + Let $g(\emptyset)$ be any point in $P$. + Suppose that $g(s) \in P$ has been chosen for all $s$ of + length $\le n$. + For each $s \in \{0,1\}^{n}$ pick + $g(s) \in (a_s, b_s)$ + such that $(a_s, b_s) \cap (a_{s'}, b_{s'}) = \emptyset$ + for all $s, s'$ of length $n$, + $b_s - a_s \le \frac{1}{n^3}$ + and + $(a_{s\defon{n-1}}, b_{s\defon{n-1}}) \subseteq (a_s, b_s)$. + + For each such $s$ pick $x_s \in (a_s, b_s) \cap P$ + with $x_s \neq f(s)$. + This is possible since $P \subseteq P'$. + Now set $g(s\concat 0) \coloneqq g(s)$ + and $g(s \concat 1) \coloneqq g(x_s)$. + This finishes the construction. + + If $t \in \{0,1\}^{\omega}$, + then $(g(t\defon{n}), n < \omega)$ is a Cauchy sequence. + + By $P' \subseteq P$ we get that this sequence + converges to a point in $P$. + Define $f(t)$ to be this point. + + If $t \neq t' \in \{0,1\}^{\omega}$, + then there is some $n$ such that $t\defon{n} \neq t'\defon{n}$, + hence $f(t) \in [a_{t\defon{n}}, b_{t\defon{n}}]$ + and $f(t') \in [a_{t\defon{n}}, b_{t\defon{n}}]$ + which are disjoint. + Thus $f(t) \neq f(t')$, i.e.~$f$ is injective. +\end{proof} diff --git a/logic2.tex b/logic2.tex index 15cf247..c430f4a 100644 --- a/logic2.tex +++ b/logic2.tex @@ -3,7 +3,7 @@ \course{Logic II} \lecturer{Ralf Schindler} \assistant{Mirko Bartsch} -\author{Josia Pietsch} +\author{Mirko Bartsch, Josia Pietsch} \usepackage{logic} @@ -25,6 +25,7 @@ \newpage \input{inputs/lecture_01} +\input{inputs/lecture_02} \cleardoublepage